Análise: Sequências de Números Reais

Análise: Sequências e Séries de Números Reais, veja as demonstrações e definições. Aprenda e compartilhe!

Atenção: os tópicos de analise real postados aqui no site estão passando por modificações e correções constantemente. Ainda não estão finalizados. Qualquer contribuição é bem vinda. Meu Instagram esta disponível no menu do site.

Sequências convergentes

Uma sequência \(\left(a_{n}\right)\) é dita convergente se, à medida que o índice \(n\) cresce, ou elemento \(a_{n}\) vai se tornando arbitrariamente o próximo de um número \(L\), chamado de limite da sequência.  A proximidade entre \(a_{n}\) e \(L\) É medida pelo valor absoluto da diferença entre esses dois números: 

$$|a_{n}-L|<\varepsilon$$

Isso significa dizer que \(|a_{n}-L|\) torna-se inferior a qualquer número positivo \(\varepsilon\), Por mais pequeno que seja desde que façamos o índice \(n\) suficientemente grande. 

Definição de sequências convergentes

Dizemos que uma sequência \(a_{n}\) converge para o número \(L\), ou tem limite \(L\) se, dado qualquer número \(\varepsilon>0\), é sempre possível encontrar um número \(n_{0}\) tal que:

\(n>n_{0} \Rightarrow|a-L|<\varepsilon\)

Se uma sequência converge, escrevemos \(\lim _{n \rightarrow \infty} a_{n}=L\) ou \(\lim a_{n}=L\) ou \(a_{n} \rightarrow L\). Já se a sequência não convergir, aí dizemos que ela é divergente. 

Vamos entender melhor essa parte da definição \(|a_{n}-L|< \varepsilon\). Primeiro lembre-se que \(\varepsilon\) é um número positivo. Ele é a vizinhança em torno de \(L\),  ou seja é todos os números no intervalo \((L-\varepsilon, L+\varepsilon)\). Este intervalo pode ser denotado como \(V_{\varepsilon}(L)\). E observe também que a condição \(x \in V_{\varepsilon}(L)\) pode ser escrita de três maneiras equivalentes:

\(|x_{n} – L|<\varepsilon \)

\(\Leftrightarrow-\varepsilon<x – L<\varepsilon \)

\(\Leftrightarrow L-\varepsilon<x<L+\varepsilon\)

Assim quando definimos o limite estamos dizendo que para todo \(\varepsilon\), ou seja, 

\(|x – L|<\varepsilon \)

\(\Leftrightarrow-\varepsilon<x – L<\varepsilon \)

\(\Leftrightarrow L-\varepsilon<x<L+\varepsilon\).

Observações: 

  1. Chama-se sequência nula toda sequência que converge para zero. 
  2. Ao dizermos “dado qualquer \(\varepsilon>0\)”, está implícito que \(\varepsilon\) pode ser arbitrariamente pequeno ou seja tão pequeno quanto quisermos.
  3. Uma vez que satisfeita as condições da definição, para um certo \(\varepsilon=\varepsilon_{0}\) será satisfeita para qualquer \(\varepsilon>\varepsilon_{0}\) como muitas vezes se faz para que ela fique provado para qualquer \(\varepsilon>0\).
  4. Quanto ao número \(n_{0}\),  podemos supô-lo um inteiro positivo, portanto um índice da sequência, mas \(n_{0}\) pode não ser um inteiro como veremos mais adiante.
  5. O primeiro sinal da desigualdade na definição pode ser tanto \(>\) quanto \(\geq\), do mesmo modo que o segundo pode ser \(<\) como \(\leq\).
  6. E se é possível fazer \(\left|a_{n}-L\right|<\varepsilon\) com qualquer \(\varepsilon>0\) certamente é possível fazer \(\left|a_{n}-L\right|<\frac{\varepsilon}{2}\) portanto \(\left|a_{n}-L\right|<\varepsilon\). 
  7.  Note que também tanto faz fazer \(\left|a_{n}-L\right|<\varepsilon\) ou \(|a+n|< k \varepsilon\) onde \(k\) é uma constante positiva, pois é possível fazer  \(|a+n|< k \varepsilon\), certamente é possível fazer \(\left|a_{n}-L\right| < k(\varepsilon / k)=\varepsilon\)
  8. Se suprimimos de uma sequência \(a_{n}\) o número finito de seus termos, isso não altera em nada o fato de que a sequência converge para L,  ou diverge a nova sequência que obterá para ele ou divergir a respectivamente. Desde que reste uma infinidade de termos de modo a termos ainda uma sequência.
  9. Observe também que a determinação de \(n_{0}\) depende do \(\varepsilon\) particular que se considere, mudando-se \(n_{0}\) deve-se, em geral, mudar também o número \(\varepsilon\). Ou seja, \(\varepsilon\) pode ser dado arbitrariamente, mas uma vez por escrito não pode ser mudado até determinação de \(n_{0}\). 

Sequências limitadas 

Dizemos que uma sequência \(a_{n}\) é limitada a esquerda ou limitada inferiormente quando existe um número A tal que:

\(A \leq a_{n}, \forall n\). 

Dizemos que uma sequência \(a_{n}\) é limitada a direta ou limitada superiormente quando existe um número B tal que:

\(B \geq a_{n}, \forall n\).

Dizemos que uma sequência \(a_{n}\) é limitada quando ela for limitada tanto à esquerda quanto à direita. Ou seja existe um número M tal que:

\(\left|a_{n}\right| \leq M, \forall n\). 

Teorema 1

Toda sequência convergente é limitada. 

Demonstração

Seja \(a_{n} \rightarrow L\), então dado qualquer \(\varepsilon>0\), existe \(n_{0}\) tal que:

\(n>n_{0} \Rightarrow L-\varepsilon<a_{n}<L+\varepsilon\)

Isto nos diz que a partir de um índice \(n \geq n_{0}+1\), a sequência é limitada a direita por \(L+\varepsilon\) e limitada à esquerda por \(L-\varepsilon\). Para englobar mas a sequência inteira, basta considerar, dentre todos os números 

\(a_{1}, a_{2}, \ldots, a_{n_{0}}, L-\varepsilon, L+\varepsilon\)

Aquele que for o menor de todos, digamos, A, e aquele que é o maior de todos, digamos, B. Sendo assim temos que para todo n é válido o que: 

\(A \leq a_{n} \leq B\)

Podíamos também ter atalhado um pouco procedendo da seguinte maneira: tomando M igual ao máximo dentre os elementos abaixo  

\(\left\{|a_{1}|, \ldots,|a_{n},|L-\varepsilon|,|L+\varepsilon|\right\}\). 

Logo, \(\left|a_{n}\right| \leq M\), para todo \(n\), o que prova que a sequência é limitada. 

Teorema 2

Se uma sequência \(a_{n}\) converge para um limite \(L\), e se \(A < L < B\), então, a partir de um certo índice \(n_{0}\) \(A < a_{n} < B\).

Este teorema também é conhecido como teorema da permanência do sinal. 

Demonstração

Temos por hipótese que \(\lim a_{n}=L\), ou seja, dado \(\varepsilon>0, \exists n_{0} \in \mathbb{N}\) tal que \(n > n_{0} \Rightarrow \left|a_{n} – L\right| < \varepsilon\), o que equivale a:

$$ n>n_{0} \Rightarrow L-\varepsilon<a_{n}<L+\varepsilon \text {. } $$

Ou seja, posso pegar qualquer valor positivo para \(\varepsilon\) que sempre existirá um \(n_{0}\) que faz com que \( L-\varepsilon<a_{n}<L+\varepsilon \) aconteça. Observe que \(a_{n}\) está entre \(L-\varepsilon\) e \(L+\varepsilon\) basta fazermos com que \(L-\varepsilon\) “vire A” e que \(L+\varepsilon\) “vire B” provamos nossa tese. 

Continuando:

Como por hipótese \(A < L < B\), então temos que: 

\(L – A > 0\) 

\(B – L > 0\) 

Tomemos \( 0 < \varepsilon \leq min \left\{ L – A, B – L\right\}\).

Daí temos: 

$$ \begin{aligned} \varepsilon \leq L – A \Rightarrow-\varepsilon \geq -L+A & \Rightarrow L-\varepsilon \geq A \\ & \Leftrightarrow A \leq L-\varepsilon \end{aligned} $$

E também que:

$$\varepsilon \leq B-L \Rightarrow L+\varepsilon \leq B$$.

Assim concluímos que:

$$ A \leq L-\varepsilon<a_{n}<L+\varepsilon \leq B $$

E portanto \(\exists n_{0} \in \mathbb{N}\) tal que \(n > n_{0} \Rightarrow A < a_{n} < B\).

Corolário 2.1

Se uma sequência \(a_{n}\) converge para um limite \(L \neq 0\), então, a partir de certo índice \(n_{0}\), \(\left|a_{n}\right|>|L| / 2\).

Demonstração:

$$\text { Se } L>0 \text {, tome } A=L / 2 \text {. } $$

$$\text {Se } L<0 \text {, tome } B=L / 2 \text {. }$$

Resumindo:

\(\left(a_{n}\right)\) é limitada:

\(\Leftrightarrow \alpha \leqslant a_{n} \leqslant \beta \)

\( \Leftrightarrow \exists M>0:-M \leqslant a_{n} \leqslant M \)

\( \Leftrightarrow \exists M>0:\left|a_{n}\right| \leqslant M\)

Teorema da unicidade do limite

Se \(\displaystyle \lim_{ }x_{n} = a\) e \(\displaystyle \lim_{ }x_{n} = b\) então \(a = b\).

Demonstração

Para fazer a demonstração podemos usar uma abordagem direta e mostrar que:

se \(\displaystyle \lim_{ }x_{n} = a\) e \(\displaystyle \lim_{ }x_{n} = b\) então \(a = b\).

Ou podemos assumir que \(a \neq b\) e mostrar que isso força uma contradição e consequentemente \(a = b\).

Então, supondo por contradição que \(a \neq b\), um deles é maior do que o outro, então vamos supor, sem perda de generalidade, que \(b > a\).

Se pensarmos no que significa uma sequência convergir para um limite \(a\), vemos que seus termos ficam arbitrariamente próximos de \(a\) e, nesse caso, como a sequência também converge para \(b\) eles devem ficar arbitrariamente próximos de \(b\).

Como assumimos que \(a \neq b\) nossa sequência deve ficar arbitrariamente próxima de dois números distintos (que possuem uma distância entre eles) o que não parece fazer muito sentido não é mesmo? E é ai que encontramos a nossa contradição.

Sendo \(a \neq b\) e \(\epsilon > 0\) devemos ter então os intervalos \(\left(a \; – \; \varepsilon, a \; +\; \varepsilon \right)\) e \(\left(b \; – \; \varepsilon, b \; +\; \varepsilon \right)\) pegando um \(\varepsilon\) pequeno o suficiente esses intervalos serão disjuntos e isso ira forçar a contradição que estamos procurando mostrar.

Então a questão é: “quão pequeno deve ser o \(\varepsilon\) escolhido para forçar essa contradição?”.

Podemos começar observando que \(\varepsilon = b \;- \;a > 0\) nesse caso todos os termos da nossa sequencia estarão eventualmente a uma distância \(b\;-\;a\). O que não é um tamanho pequeno o suficiente para forçar uma contradição, os termos da nossa sequência podem muito bem estar a uma distância \(b\;-\;a\).

Mas se pegarmos \(\varepsilon = \frac{b \;-\; a}{2} > 0\) irá funcionar pois pela definição de convergência os termos da nossa sequencia eventualmente estarão a uma distancia menor do que

\(\varepsilon = \frac{b\; -\; a}{2} > 0\) de \(a\)

e também a uma distancia menor do que

\(\varepsilon = \frac{b \;- \;a}{2} > 0\) de \(b\).

Pela definição de limite de uma sequência temos que se \(\varepsilon = \frac{b \;- \;a}{2} > 0\) então

\(\exists n_{1} \in \mathbb{N}, \;\forall n > n_{1}\)

\(\Rightarrow |a_{n} \; -\; a| < \frac{b\; -\; a}{2}\)

e

\(\exists n_{2} \in \mathbb{N}, \;\forall n > n_{2}\)

\(\Rightarrow |a_{n} \; -\; b| < \frac{b\; -\; a}{2}\)

Garantindo ambas as restrições conseguimos a nossa contradição, mas qual valor \(N\) devemos passar para garantir ambas as restrições.

Como precisamos ter certeza de que passamos dos termos \(n_{1}\) e \(n_{2}\). Nesse caso consideramos \(n = max\left\{n_{1}, n_{2}\right\}\) então \(\forall n > n_{0}\), com isso garantimos que todos os termos da nossa sequência são maiores do que \(n_{1}\) e \(n_{2}\), garantindo ambas as inequações.

Então temos que:

\(|a_{n}\; -\; a | < \frac{b \; – \; a}{2}\).

Que é equivalente a:

\(-\frac{b\; – \; a}{2} < a_{n}\; -\; a < \frac{b\;-\;a}{2}\) \(\Rightarrow\) \(\frac{a\; – \; b}{2} < a_{n} \;- \;a < \frac{b\;-\;a}{2}\) \(\Rightarrow\) \(\frac{a\; + \; b}{2} < a_{n} < \frac{b\; + \;a}{2}\).

E também que:

\(|a_{n}\; – \;b | < \frac{b \; – \; a}{2}\).

Que é equivalente a:

\(-\frac{b\; – \; a}{2} < a_{n}\; -\; b < \frac{b\;-\;a}{2}\) \(\Rightarrow\)

\(\Rightarrow\) \(\frac{a\; – \; b}{2} < a_{n}\; – \;b < \frac{b\;-\;a}{2}\)

\(\Rightarrow\) \(\frac{a\; + \; b}{2} < a_{n}\).

Assim vemos que \(a_{n} < \frac{b\; + \;a}{2}\) e \(\frac{a\; + \; b}{2} < a_{n}\) o que não é possível.

Dessa forma \(a\) deve ser igual a \(b\).

Segunda forma de demonstrar a unicidade do limite

Seja \(\displaystyle \lim_{ }x_{n} = a\) uma sequencia convergente (pois somente sequências convergentes possuem limites). Dado qualquer número real \(b\), sendo \(b \neq a\), mostraremos que não se tem \(\displaystyle \lim_{ }x_{n} = b\).

Para isso tomaremos \(\varepsilon = \frac{|b \; – \; a|}{2}\).

Vemos que \(\varepsilon > 0\) e notemos ainda que os intervalos \(\left(a \; – \; \varepsilon, a \; +\; \varepsilon \right)\) e \(\left(b \; – \; \varepsilon, b \; +\; \varepsilon \right)\) são disjuntos.

Se existisse

\(x \in \left(a \; – \; \varepsilon, a \; +\; \varepsilon \right) \cap \left(b \; – \; \varepsilon, b \; +\; \varepsilon \right) \)

teríamos

\(|a\; – \; x| < \varepsilon\) e \(|x \: – \; b| < \varepsilon\),

donde

\(|a \; – \; b| \leq |a \; – \; x| + |x \; – \; b| < 2 \varepsilon \)

\(= |a \; – \; b|\),

um absurdo.

Como \(\displaystyle \lim_{ }x_{n} = a\), existe \(n_{0} \in \mathbb{N}\) tal que \(n > n_{0} \Rightarrow x_{n} \in \left(a \; – \; \varepsilon, a \; +\; \varepsilon \right)\) e, portanto \(x_{n} \notin \left(b \; – \; \varepsilon, b \; +\; \varepsilon \right)\) para todo \(n > n_{0}\). Logo não é \(\displaystyle \lim_{ }x_{n} = b\).

Terceira forma de demonstrar a unicidade do limite

Demonstração: Seja \(\displaystyle \lim_{ }x_{n} = a\). Dado \(b \neq a\) podemos tomar \(\varepsilon > 0\) tal que os intervalos abertos \(I = \left(a \;-\; e, a + e \right)\) e \(J = \left(b \;-\; e, b + e \right)\) sejam disjuntos. Existe \(n_{0} \in \mathbb{N}\) tal que \(n > n_{0}\) implica \(x_{n} \in J\). Então, para todo \(n > n_{0}\), temos \(x_{n} \notin J\). Logo não é \(\displaystyle \lim_{ }x_{n} = b\).

Operações com limites 

Teorema

Sejam \(a_{n}\) e \(b_{n}\) duas sequências convergentes, com limites \(a\) e \(b\) respectivamente. Então, \(a_{n} + b_{n}\), \(a_{n} \cdot b_{n}\) e \(k \cdot b_{n}\), onde \(k\) uma constante qualquer, são sequências convergentes, além do que,

\(a) \operatorname{lim}\left(a_{n}+b_{n}\right)=\lim a_{n}+\operatorname{lim} b_{n}=a+b\);

Demonstração

Devemos provar que dado \(\varepsilon>0, \exists n_{0} \in \mathbb{N}\) tal que \(n>n_{0} \Rightarrow\left|\left(a_{n}+b_{n}\right)-(a+b)\right|<\varepsilon\). Temos por hipotese que \(a_{n} \rightarrow a\) e \(b_{n} \rightarrow b\).

Assim, dado \(\frac{\varepsilon}{2}>0\), existe \(n_{1}\) e \(n_{2} \in \mathbb{N}\) tal que

\(n > n_{1} \Rightarrow\left|a_{n}-a\right|<\varepsilon / 2\),

\(n > n_{2} \Rightarrow\left|b_{n}-b\right|<\varepsilon / 2\).

Portanto,

$$ \left|\left(a_{n}+b_{n}\right)-(a+b)\right|<\varepsilon \Leftrightarrow \lim \left(a_{n}+b_{n}\right)=a + b $$

Agora note que:

$$ \left|\left(a_{n}+b_{n}\right)-(a+b)\right|=\left|\left(a_{n}-a\right)+\left(b_{n}-b\right)\right|\leq \left|a_{n}-a\right|+\left|b_{n}-b\right| $$

Logo, tomando \(n_{0} \geq \max \left\{n_{1}, n_{2}\right\}\) temos que
$$ \left.n>n_{0} \Rightarrow\left|\left(a_{n}+b_{n}\right)-\right| a+b\right) \mid \leq \frac{\varepsilon}{2}+\frac{\varepsilon}{2}=\varepsilon$$

\(b) \lim \left(k a_{n}\right)=k\left(\operatorname{lim} a_{n}\right)=k a_{1}\) em particular, \(k=-1\) nos dá \(a_{n} \rightarrow a_{n} \Rightarrow-a_{n} \rightarrow-a_{i}\)

Demonstração

Inicialmente, é obvio que a igualdade vale para \(k = 0\), pois

\(\lim \left(0 \cdot a_{n}\right)=\lim (0)=0=0 \cdot a\)

Vamos então considerar \(k \neq 0\).

Devemos provar que dado \(\varepsilon > 0\), existe \(n_{0} \in \mathbb{N}\) tal que

\(n>n_{0} \Rightarrow\left|k \cdot a_{n}-k \cdot a\right|<\varepsilon\).

Temos por hipótese que \(a_{n} \rightarrow a\).

Então, dado \(\frac{\varepsilon}{|k|}>0\), existe \(n_{0} \in \mathbb{N}\).
$$ n > n_{0} \Rightarrow\left|a_{n}-a\right|<\frac{\varepsilon}{|k|} \text {. } $$ Agora note que: $$ \left|k \cdot a_{n}-k \cdot a\right|=\left|k \cdot\left(a_{n}-a\right)\right|=|k| \cdot\left|a_{n}-a\right|$$ então se \(n>n_{0}\) temos:
$$ \left|k \cdot a_{n}-k \cdot a\right|=|k| \cdot\left|a_{n}-a\right|<|k| \cdot \frac{\varepsilon}{|k|}=\varepsilon \text {. } $$

Portanto, \(n>n_{0} \Rightarrow\left|k \cdot a_{n}-k \cdot a\right|<\varepsilon\), ou seja
$$ \lim \left(k \cdot a_{n}\right)=k \cdot a $$

\(c) \lim \left(a_{n} b_{n}\right)=\left(\operatorname{lim} a_{n}\right)\left(\operatorname{lim} b_{n}\right)=a b\)

Demonstração:

Devemos provar que, dado \(\varepsilon>0, \exists n_{0} \in \mathbb{N}\) tal que \(n>n_{0} \Rightarrow \left|a_{n} b_{n}-a b\right|<\varepsilon\).

Note que:

\begin{aligned} \left|a_{n} b_{n}-a b\right| &=\left|a_{n} b_{n}-a b_{n}+a b_{n}-a b\right| \\ &=\left|\left(a_{n}-a\right) b_{n}+a\left(b_{n}-b\right)\right| \\ & \leq \left|\left(a_{n}-a\right) \cdot b n\right|+\left|a \cdot\left(b_{n}-b\right)\right| \\ &=\left|a_{n}-a\right| \cdot\left|b_{n}\right|+|a| \cdot\left|b_{n}-b\right| \end{aligned}

Como,

\(a_{n} \rightarrow a, n>n_{1}\) \( \Rightarrow \left|a_{n} – a\right|<\frac{\varepsilon}{2M} \)

\(b_{n} \rightarrow b, n>n_{2} \) \(\Rightarrow\left|b_{n}-b\right|<\frac{\varepsilon}{2|a|}\)

$$ n>n_{3} \Rightarrow\left|b_{n}\right| \leq M $$

Então \(n>\max \left\{n_{1}, n_{2}, n_{3}\right\}\) implica que

\(\left|a_{n} b_{n}-a b\right| = \left|a_{n}-a\right| \cdot\left|b_{n}\right|+|a| \cdot\left|b_{n}-b\right|<\frac{\varepsilon}{2 M} \cdot M+|a| \frac{\varepsilon}{2|a|}\)

\(=\frac{\varepsilon}{2}+\frac{\varepsilon}{2}=\varepsilon \)

Concluímos então que:

\(n>n_{0} \Rightarrow\left|a_{n} b_{n}-a b\right|<\varepsilon \Leftrightarrow \lim \left(a_{n} b_{n}\right)=a b \).

\(d)\) Se, além das hipóteses acima, \(b \neq 0\), então existe o limite de \(a_{n}\) / \(b_{n}\), igual a \(a/b\).

Demonstração

Qual o limite da raiz enésima de a?

Neste exemplo usamos operações com limites e a desigualdade de Bernoulli.

Dado um número \(a>0, \sqrt[n]{a} \rightarrow 1\). Isso é evidente se \(a=1\), quando a sequência é constantemente igual a 1. Ou seja, o limite da raiz enésima de a é 1. Agora vamos ver como provar isso.

Demonstração

\(a>0, \sqrt[n]{a} \rightarrow 1\)

i) Se \(a = 1\), então \(\sqrt[n]{1} = 1,1,1, \ldots \rightarrow 1\);
ii) se \(a>1 \Rightarrow \sqrt[n]{a}>1 \Rightarrow \sqrt[n]{a}=1+h_{n}, \;h_{n}>0\).

Note que,
$$ a=\left(1+h_{n}\right)^{n}$$ e $$h_{n}= |\sqrt{a}-1|.$$

Por Bernoulli \(a=\left(1+h_{m}\right)^{n} \geq 1 + nh_{n}>n h_{n}\) e sabemos que \(h_{n}<\frac{a}{n}\). Sendo assim, $$ |\sqrt{a}-1|<\frac{a}{n} $$ O que vale \(\forall \varepsilon>0\) dado, pois basta tomar \(n > \frac{a}{\varepsilon}\)

iii) \(0< a<1 \Rightarrow \frac{1}{a}>1\), daí \(\lim \frac{1}{\sqrt[n]{a}} \rightarrow 1\).

do resultado anterior.

Qual o limite da raiz enésima de n?

$$\sqrt[n]{n} \rightarrow 1$$

Como você pode ver o limite da raiz enésima de \(n\) tende a \(1\). Agora precisamos provar essa afirmação.

Sequências monótonas

Teorema 1

Toda sequência monótona e limitada é convergente.

Demonstração

Consideremos, para fixar as ideias, uma sequência não decrescente \(\left(a_{n}\right)\) (portanto, limitada inferiormente pelo elemento \(a_{1}\) ). A hipótese de ser limitada significa que ela é limitada superiormente; logo, seu conjunto de valores possui supremo \(S\). Vamos provar que esse número \(S\) é o limite de \(a_{n}\).
Dado \(\varepsilon>0\), existe um elemento da sequência, com um certo índice \(n_{0}\), tal que \(S-\varepsilon n_{0}\), de sorte que

$$ n > n_{0} \Rightarrow S – \varepsilon < a_{n} < S + \varepsilon $$

que é o que desejávamos demonstrar.

Sequencias monótonas

\(\left(a_{n}\right)\) crescente \(\Rightarrow a_{1}a_{2}>\ldots>a_{n}>\ldots\)
\(\left(a_{n}\right)\) não decrescente \(\Rightarrow a_{1} \leqslant a_{2} \leqslant \cdots \leqslant a_{n} \leqslant \ldots\)
\(\left(a_{n}\right)\) não crescente \(\Rightarrow a_{1} \geqslant a_{2} \geqslant \cdots \geqslant a_{n} \geqslant \ldots\)

Estas são as sequencias monótonas.

Teorema: Toda sequência monótona e limitada é convergente.

Demonstração

Para fixar as ideias, vamos considerar uma sequência não decrescente \(\left(a_{n}\right)\), portanto, limitada inferiormente pelo elemento \(\left.a_{1}\right)\).

Da hipótese de ser limitada vem que \(a_{n}\) é limitada superiormente; logo, seu conjunto de valores possui supremo \(S\).

Como \(S\) é supremo, dado \(\varepsilon>0\), existe \(n_{0} \in \mathbb{N}\) tal que \(S-\varepsilon<a_{n_{0}}<S\). Como \(a_{n}\) é não decrescente, então \(n > n_{0} \Rightarrow a_{n_{0}} \leq a_{n}\). Dessa forma:

Vamos provar que esse número \(S\) é o limite de \(a_{n}\).

\begin{aligned} & n>n_{0} \Rightarrow S-\varepsilon<a_{n_{0}} \leqslant a_{n} \leqslant S<S+\varepsilon \\ &n>n_{0} \Rightarrow S-\varepsilon<a_{n}<S+\varepsilon \\ &n>n_{0} \Rightarrow-\varepsilon<a_{n}-S<+\varepsilon \\ &n>n_{0} \Rightarrow\left|a_{n}-S\right|<\varepsilon \text {, ou seja } \lim a_{n}=S \end{aligned}

que é o que desejávamos demonstrar.

A demonstração do teorema no caso de uma sequencia não crescente é análoga.

Subsequências

Quando eliminamos um ou vários termos de uma dada sequência, obtemos o que se chama uma “subsequência” da primeira.

\begin{aligned} &\left(a_{n}\right)=(1,2,3,4, \ldots, n, \ldots) \\ &\left(b_{n}\right)=(2,4,6, \ldots, 2 n, \ldots)=\left(a_{2 n}\right) \\ &\left(c_{n}\right)=(1,3,5, \ldots, 2 n-1, \ldots)=\left(a_{2 n-1}\right) \\ &\left(x_{n}\right)=(2,3,4,5, \ldots)=\left(a_{n+1}\right) \end{aligned}

Definição: Dada uma sequência \(a_{n}\), uma subsequência de \(a_{n}\) é outra sequência, obtida a partir de \(a_{n}\) fazendo-se uma restrição dos índices a um conjunto infinito \(\mathbb{N}\prime \subset \mathbb{N}\). Ou seja, uma subsequência de \(a_{n}\) é uma sequência \(b_{j}\) = \(a_{nj}\), onde \((b j)=\left(a_{n j}\right), \text { onde } n_{1}<n_{2}<n_{3}<\ldots \text { e }\left\{n_{1}, n_{2}, n_{3}, \ldots\right\}=\mathbb{N}^{\prime} \subset \mathbb{N}\)

Observação: \(\left(a_{n}\right)=\left(a_{1}, a_{2}, a_{3}, a_{4}, a_{5}, \ldots\right) \longrightarrow \) Sequência.

\(\left(a_{n j}\right)=\left(a_{n_{1}}, a_{n_{2}}, a_{n_{3}}, a_{n_{11}}, a_{n_{1}}, \ldots\right) \longrightarrow\) Subsequência.

Note que \(1 \leqslant n_{1},\; 2 \leq n_{2},\; \ldots,\; j \leq n_{j},\; \ldots\)

e, além disso, \(j<n_{j} \Rightarrow k<n_{k},\forall \;k > j\).

Exemplo

A sequência \(\left(a_{n}\right)=(-1)^{n}(1+1 / n)\) é divergente pois, se \(n\) é par ela tende para 1 e se \(n\) é impar ela tende para \(-1\).

Porém todas as subsequências formadas por termos pares são convergentes \(\left(a_{2 n}\right),\left(a_{4 n}\right)\), \(\left(a_{6 n}\right)\) etc., todas convergem para 1; e todas as subsequências formadas por termos ímpares são convergentes \(\left(a_{2 n-1}\right),\left(a_{4 n-1}\right),\left(a_{6 n-1}\right)\) etc., todas convergindo para \(-1\).

Mas tem também subsequências divergentes, como \(\left(a_{n^{2}}\right)=\left(a_{1}, a_{4}, a_{9}, a_{16}, \ldots\right)=(-2,5 / 4,-10 / 9,17 / 16,\).

Teorema

Se uma sequência \(\left(a_{n}\right)\) converge para um limite. \(L\), então toda sua subsequência \(\left(a_{n_{j}}\right)\) também converge para \(L\).

Vamos análisar o teorema e a sua contrapositiva.
Se \(\lim a_{n}=L\), então, \(\lim a_{n j}=L\), para toda subsequência \(\left(a_{n_{j}}\right)\). Ou seja, \(a_{n} \rightarrow L \Rightarrow a_{n j} \rightarrow L, \forall a_{n j}\)

Contrapositiva: Se existissem \(\left(a_{n_{j}}\right)\) e \(\left(a_{m_{j}}\right)\) tais que \(a_{n y} \rightarrow L+a_{m_{j}} \rightarrow M\), com \(L \neq M\), então não existe \(lim a_{n}\), ou seja, \(a_{n}\) é divergente.

Demonstração

Por definição temos que \(a_{n} \rightarrow L\) portanto, dado qualquer \(\varepsilon>0\) existe \(n_{0}\) tal que \(n>n_{0} \Rightarrow\left|a_{n}-L\right|<\varepsilon\).

Como vimos acima, \(n_{j} \geq j > n_{0}\), o que implica em \(\left|a_{n_{j}}-L\right|<\varepsilon\).

Concluímos então que: dado \(\varepsilon > 0, \exists n_{0} \in \mathbb{N}\) tal que \(j > n_{0} \Rightarrow n_{j}>n_{0} \Rightarrow \left|a_{nj} – L \right| < \varepsilon \Rightarrow lim a_{nj} = L\).

Limites infinitos

Definição

Diz-se que a sequencia \(\left(a_{n}\right) \) diverge (ou tende) para \(+\infty\) e escreve-se \(\lim a_{n}=+\infty\) ou \(\lim a_{n}=\infty\) se, dado qualquer número positivo \(k\), existe \(n_{0}\) tal que \(n>n_{0} \Rightarrow a_{n}>k\). Analogamente, \(\left(a_{n}\right)\) diverge (ou tende) para \(-\infty\) se, dado qualquer número negativo \(k\), existe \(n_{0}\) tal que \(n>n_{0} \Rightarrow a_{n}<k\); neste caso, escreve-se \(\lim a_{n}=-\infty\).

Exemplo

Prove que: \(a_{n}=n \longrightarrow+\infty\).

Solução:

\(\lim a_{n}=\lim n=+\infty\)

Dado \(k>0\), basta tomar \(n_{0} \geq k\), daí \(n>n_{0} \Rightarrow n>K \Rightarrow a_{n}>K\).

Portanto \(\lim a_{n}=+\infty\).

Prove que: \(a_{n}=n^{2}+1 \longrightarrow+\infty\).

Solução: Dado \(k>0\), basta tomar \(n_{0} \geq \sqrt{k}\), daí

\( n>n_{0} \Rightarrow n>\sqrt{k}>0 \Rightarrow n^{2}>(\sqrt{k})^{2} \Rightarrow n^{2}>K \)

\(\Rightarrow n^{2}+1>n^{2}>K \Rightarrow n^{2}+1>k \Rightarrow a_{n}>K\)

Portanto, \(\lim a_{n}=+\infty\).

Prove que: \(a_{n}= \sqrt{n} \longrightarrow+\infty\).

Dado \(k>0\), basta tomar \(n_{0} \geq K^{2}\), daí

\( n>n_{0} \Rightarrow n>K^{2}>0 \Rightarrow \sqrt{n}>\sqrt{K^{2}} \)
\(\Rightarrow \sqrt{n}>k \Rightarrow a_{n}>k\).

Logo, \(\lim a_{n}=+\infty\).

Prove que: \(a_{n}=3-n^{2} \longrightarrow-\infty\)

Dado \(k<0\), basta tomar \(n_{0} \geq \sqrt{3-k}\), daí

\(n>n_{0} \Rightarrow n>\sqrt{3-k} \Rightarrow n^{2}>(\sqrt{3-k})^{2} \Rightarrow n^{2}>3-k \)

\(\Rightarrow-n^{2}<k-3 \Rightarrow 3-n^{2}<3+k-3=k \)

\(\Rightarrow 3-n^{2}<k\)

Logo, \(\lim a_{n}=-\infty\).

Teorema (operações com limites infinitos)

a) \(a_{n} \rightarrow+\infty \Leftrightarrow-a_{n} \rightarrow-\infty\).

b) Seja \(\left(a_{n}\right)\) uma sequência não limitada. Sendo não decrescente, ela tende \(a+\infty\); e sendo não crescente, ela tende \(a-\infty\).

c) Se \(\lim a_{n}=\pm \infty\), então \(1 / a_{n}\) tende a zero.

d) Se \(\lim a_{n}=0\), então \(1 / a_{n}\) tende a \(+\infty\) se \(a_{n}>0 /\) e tende \(a-\infty\) se \(a_{n}<0\).

e) Se \(\left(b_{n}\right)\) é uma seqüência limitada e \(a_{n} \rightarrow+\infty\) ou a \(-\infty\), então a sequência \(\left(a_{n}+b_{n}\right)\) tende a \(+\infty\) ou a \(-\infty\) respectivamente.

f) Se \(a_{n} \rightarrow+\infty\) e \(b_{n} \geq c\), onde c é um número positivo, então \(a_{n} b_{n} \rightarrow+\infty\). (Em particular, \(a_{n} \rightarrow+\infty\) e \( b_{n} \rightarrow+\infty \Rightarrow a_{n} b_{n} \rightarrow+\infty\).) Formule e demonstre as outras possibilidades: \(a_{n} \rightarrow+\infty\) e \(b_{n} \leq c<0, a_{n} \rightarrow-\infty\) e \(b_{n} \geq c>0, a_{n} \rightarrow-\infty\) e \(b_{n} \leq c<0\)

g) Se \(a_{n} \rightarrow+\infty\) e \(a_{n} \leq b_{n}\), então \(b_{n} \rightarrow + \infty\).

Exemplo

\(a_{n}=a^{n}, a>1\)

\(\left(2^{n}\right)=(2,4,8,16, \ldots)\)

Vamos mostrar que \(a_{n} \longrightarrow+\infty\).

De fato, note que \(a>1 \Rightarrow 0<1 / a<1\).

Pelo ex. 10 da pág. 55 , temos que \(\left(\frac{1}{a}\right)^{n}=\frac{1}{a^{n}} \longrightarrow 0\).

Daí, pelo ifem \((b)\) do teorema de operações com limites infinitos temos que \(\frac{1}{1 / a^{n}}=a^{n} \longrightarrow \infty\).

Podemos mostrar usando a desigualdade de Bernoulli

Como por hipótese \(a>1\), então existe \(h>0\) tal que \(a=1+h\). Dessa forma, pela desigualdade de Bernoulli, temos:

$$ a^{n}=(1+h)^{n}>1+n h>n h.$$

Então, dado \(k>0\), basta tomar \(n_{0} \geq \frac{k}{h}\), pois daí,

\begin{aligned} n>n_{0} & \Rightarrow n>\frac{k}{h} \Rightarrow n h>\frac{k}{h} \cdot h=k \\ & \Rightarrow 1+n h>k \Rightarrow(1+h)^{n}>k \\ & \Rightarrow a^{n}>k \Rightarrow \lim a^{n}=+\infty \end{aligned}

Ou ainda, podemos provar usando \(log\). Ou seja, dado \(k>0\), basta tomar \(n_{0} \geq \frac{\log k}{\log a}\), pois

\(n>n_{0} \Rightarrow n>\frac{\log k}{\log a}\)

Temos que \(log a\) é positivo pois \(a > 1\), então

\(\Rightarrow n \cdot \log a>\log k \)

Aplicando as propriedades do logaritmo

\(\Rightarrow \log a^{n}>\log k\)

Sabendo que a função log é crescente

\(\Rightarrow a^{n}>k \Rightarrow a^{n} \rightarrow+\infty\)

Mais um exemplo

Seja a sequência \(a_{n}=n^{k}\). Daí, \(a_{n}=\underbrace{n \cdot n \cdot \ldots \cdot n}_{k \text { tatores }}=n^{k}\)

Como \(n \longrightarrow \infty\), então pelo teorema 2.16(f), temos que \(a_{n}=n^{k} \longrightarrow \infty\).

Assim, temos que se \(a>1, a^{n} \longrightarrow \infty) e ( k \in \mathbb{N}, \quad n^{k} \longrightarrow \infty \text {. }\)

Porém, é possével mostrar que \(a^{n}\), \(a>1\), tende a infinito mais rapidanvente do que \(n^{k}\).

Elon, exemplo, 8 e 9 página 29.

Sabemos que (exemplo 8 Elon) se \(x_{n}>0\) e \(\lim \left(\frac{x_{n+1}}{x_{n}}\right)=a<1\), então \(\lim x_{n}=0\). Dessa forma, vemos (exemplo 9 Elon) que:

$$ \begin{aligned} x_{n}=\frac{n^{k}}{a^{n}} \Rightarrow \frac{x_{n+1}}{x_{n}} &=\frac{(n+1)^{k} / a^{n+1}}{n^{k} / a^{n}}=\frac{(n+1)^{k}}{a^{n+1}} \cdot \frac{a^{n}}{n^{k}} \\ &=\left(\frac{n+1}{n}\right)^{k} \cdot \frac{1}{a}=\left(1+\frac{1}{n}\right)^{k} \cdot \frac{1}{a} \end{aligned} $$

Assim, note que \(x_{n}>0\) e

$$ \lim \left(\frac{x_{n+1}}{x_{n}}\right)=\lim \left(1+\frac{1}{n}\right)^{k} \cdot \frac{1}{a}=1^{k} \cdot \frac{1}{a} $$

$$ =1 . \frac{1}{a}=\frac{1}{a} \text {, pois } a>1. $$

Conclusão, \(\lim x_{n}=0\), isto é: \(\lim \frac{n^{k}}{a^{n}}=0\).

Teorema de Bolzano-Weierstrass

Toda sequência limitada \(a_{n}\) admite subsequência convergente.

\begin{aligned} \left(a_{n}\right) \text { é limitada } \Rightarrow & \exists L, M \in \mathbb{R} \ e \ \exists \ n_{0} \in \mathbb{N}, \text{ tal que } \\ & n>n_{0} \Rightarrow L \leq a_{n} \leq M \\ & \text { ou } \\ & \exists K>0 \text { e } \exists n_{0} \in \mathbb{N} \text{ tal que } \\ & n>n_{0} \Rightarrow\left|a_{n}\right| \leq K . \end{aligned}

\(\left(a_{n}\right)\) é convergente \(\Leftrightarrow\) dado \(\varepsilon>0, \exists x_{0} \in \mathbb{N} {\text { tal que}} n>n_{0} \Rightarrow\left|a_{n}-L\right|<\varepsilon \text {. }\)

Demonstração

Seja \(\left(a_{n}\right)\) limitada. Entāo \(a_{n} \in I\), intervalo fechado, \(|I|=c\).

\begin{aligned} &\left|I_{I}\right|=c \\ &\left|I_{1}\right|=c / 2=\frac{c}{2^{1}} \\ &\left|I_{2}\right|=\frac{c}{4}=\frac{c}{2^{2}} \\ &\left|I_{3}\right|=\frac{c}{8}=\frac{c}{2^{3}} \\ &\left|I_{j}\right|=\frac{c}{j} \end{aligned}

$$ \left|I_{n}\right|=\frac{c}{2^{n}} \longrightarrow 0 \text {. } $$

Pelo teorema dos intervalos caixados existe \(L \in \mathbb{R} \text{ tal que} L \in I_{n}, \forall n \in \mathbb{N}\). Vamos construir uma subsequencia \(\left(a_{n_{j}}\right)\) de \(\left(a_{n}\right)\) com \(a_{n_{j}} \rightarrow L\).

Basta tomar \(a_{n_{1}} \in I_{1}, a_{n_{2}} \in I_{2}, \ldots, a_{n_{j}} \in I_{j}, \ldots\), de tal modo que \(n_{1}<n_{2}<n_{3}<\ldots<n_{j}<\ldots\)

Afirmação: \(\lim a_{n_{j}}=L\).

De fato, dado \(q>0, \exists n_{0} \in \mathbb{N}\) tal que \(\frac{c}{2^{n}}<\varepsilon\).

Mas, \(\left|I_{n_{0}}\right|=\frac{c}{2^{n_{0}}}<\varepsilon\), então \(m>n_{0} \Rightarrow\left|I_{m}\right|<\varepsilon\).

Ou seja, \(m>n_{0} \Rightarrow I_{m} \subset(L-\varepsilon, L+\varepsilon)\), pois \(L \in I_{m}\).

Como visto anteriormente, \(n_{j} \geq j\). Daí, se \(j>n_{0}\) então \(n_{j}>n_{0}\), logo \(I_{n_{j}} \in (L-\varepsilon, L+\varepsilon)\) implica em \(a_{n j} \in (L-\varepsilon, L+\varepsilon) \Leftrightarrow L-\varepsilon<a_{n_{j}}<L+\varepsilon \Leftrightarrow -\varepsilon<a_{n_{j}} – L<+\varepsilon \Leftrightarrow |a_{n_{j}} – L | < \varepsilon\).

Portanto, \(a_{n_{j}} \longrightarrow L\)

Critério de convergência de sequencias de Cauchy.

Teorema: Critério de Cauchy

\(\left(a_{n}\right)\) é convergente \(\Leftrightarrow\) dado \(\varepsilon>0, \exists n_{0} \in \mathbb{N}\) tal que

$$m, n>n_{0} \Rightarrow\left|a_{n}-a_{m}\right|<\varepsilon \text {. } $$

Formulação equivalente:

$$ (a_{n}) \in \mathbb{N}, n>n_{0} \Rightarrow\left|a_{n}-a_{n+p}\right|<\varepsilon .$$

Demonstração

\((\Rightarrow)\)

Seja \(\left(a_{n}\right)\) uma sequência convergente, isto é \(\exists L \in \mathbb{R}\) tal que \(\lim a_{n}=L\). Da definição de sequência convergente temos que, dado \(\varepsilon>0, \exists n_{0} \in \mathbb{N}\) tal que \(n>n_{0} \Rightarrow |a_{x}-L|<\varepsilon\) e \(m>n_{0} \Rightarrow |a_{x}-L|<\varepsilon\)

Daí note que:

\begin{aligned} \left|a_{n}-a_{m}\right| &=\left|a_{n}-L+L-a_{m}\right| \leqslant\left|a_{n}-L\right|+\left|L-a_{m}\right| \\ &=\left|a_{n}-L\right|+\left|a_{m} – L\right| \end{aligned}

Assim se,

\begin{aligned} &m, n>n_{0} \Rightarrow\left|a_{n}-a_{m}\right| \leqslant\left|a_{n}-L\right|+\left|a_{m}+L\right|<\frac{\varepsilon}{2}+\frac{\varepsilon}{2}=\varepsilon \\ &m, n>n_{0} \Rightarrow\left|a_{n}-a_{m}\right| \leqslant \varepsilon . \end{aligned}

\((\Leftarrow)\)

Por hipótese, temos que dado \(\varepsilon>0\), existe \(n_{0} \in \mathbb{N}\) tal que \(m, n>n_{0},\) então \(\left|a_{n}-a_{m}\right|<\varepsilon\).

Fazendo na hipótese acima \(m=n_{0}+1>n_{0}\), então \(n>n_{0}\), então \(\left|a_{n}-a_{n_{0}+1}\right|<\varepsilon \Leftrightarrow-\varepsilon<a_{n}-a_{n_{0}+1}<+\varepsilon\)

$$ \Leftrightarrow \underbrace{a_{n_{0}+1}-\varepsilon}_{L \in R}<a_{n}<\underbrace{a_{n_{0}+1}+\varepsilon}_{M \in \mathbb{R}}\> $$

Ou seja,

\(n>n_{0} \Rightarrow L<a_{n}<M\), isto é \(a_{n}\)

É limitada. Pois isto implica que:

$$\left|a_{n}\right| \leqslant K, \forall n \in \mathbb{N}$$

onde,

\begin{equation} K=\max \left\{\left|a_{1}\right|,\left|a_{2}\right|, \ldots,\left|a_{n_{0}}\right|,\left|a_{n_{0}+1}-\varepsilon\right|,\left|a_{n_{0}+1}+\varepsilon\right|\right\}\end{equation}

Logo, pelo teorema de Bolzano-Weierstrass, \(\left(a_{n}\right)\) possui subsequência \(\left(a_{n_{j}}\right)\) convergente \(a_{n_{j}} \rightarrow L\).

\begin{aligned} &\exists j_{1} \in \mathbb{N} j>j_{1} \text{ tal que } \Rightarrow |a_{n}-L|<\varepsilon . \\ &\exists j_{2} \in \mathbb{N} \text{ tal que } n_{i_{2}}>n_{0} . \end{aligned}

Segue que:

\(j \geqslant \max \left\{j_{1}, j_{2}\right\} \Rightarrow\left|a_{n_{j}}-L_{0}\right|<\varepsilon\) e \(n_{j}>n_{0} .\)

agora note que:

$$ \left|a_{n}-L\right|=\left|a_{n}-a_{n j}+a_{n j}-L\right| \leqslant\left|a_{n}-a_{n j}\right|+\left|a_{n j}-L\right| \text {, } $$

logo, se além de \(n_{j}>n_{0}\) tivermos também \(n>n_{0}\) :

$$ \left|a_{n}-L\right| \leqslant\left|a_{n}-a_{n_{j}}\right|+\left|a_{n j}-L\right|<\varepsilon+\varepsilon=2 \varepsilon \text {. } $$

Portanto, (n>n_{0} \Rightarrow\left|a_{n}-L\right|<2 \varepsilon), ou seja, (\lim a_{n}=L_{1}). Ou seja (a_{n}) é convergente.

Was this helpful?

0 / 0

Deixe um comentário 0

Your email address will not be published.


Compartilhe

[amount] estão lendo esse conteúdo agora.

Algumas equações são muito grande para ser visualizadas em dispositivos móveis. Para uma leitura melhor use o celular na horizontal. 

Inscreva-se no canal clicando aqui